Sie sind auf Seite 1von 15

Problem 6.

4 Determine the axial forces in the mem-


A 2 kN
bers of the truss.
0.3 m B

0.4 m

0.6 m 1.2 m

Solution: First, solve for the support reactions at B and C, and


then use the method of joints to solve for the forces in the members. A 2 kN

0.3 m B
BY A
2 kN
0.3 m 0.4 m
BX
C
0.4 m
0.6 m 1.2 m
0.6 m 1.2 m
7
CY tan =
12
 (BC = 0.961 kN)
Fx : Bx + 2 kN = 0
 = 30.26
Fy : B y + Cy = 0

 Fx = BC cos + AC cos = 0
+ MB : 0, 6Cy (0, 3)(2 kN) = 0

Solving, Bx = 2 kN Cy = 1 kN By = 1 kN Fy = BC sin + AC sin + 1 = 0
Joint B:
Solving, we get AC = 0.926 kN
y We have AB = 2.839 kN (T )
BY (1 kN)
BC = 0.961 kN (C)

3 AC = 0.926 kN (C)
AB
BX (2 kN) Check: Look at Joint A
18 x
6
y
4
BC 2 kN

 x
Fx : AB cos + BC cos 2 = 0

Fy : AB sin BC sin 1 = 0 AB

Solving, we get AB = 2.839 kN, BC = 0.961 kN AC


Joint C:


AC Fx : AB cos AC cos + 2 = 0
BC 7 
Fy : AB sin AC sin = 0

12 Substituting in the known values, the equations are satised: Check!

1 kN

Evisdom
Problem 6.9 The truss shown is part of an airplanes
internal structure. Determine the axial forces in mem- 8 kN 14 kN

bers BC, BD, and BE. C E G


A H

300 mm

Solution: First, solve for the support reactions and then use the B F
method of joints to solve for the reactions in the members. D

8 kN 14 kN 400 mm 400 mm 400 mm 400 mm

0.4 m 0.4 m 0.4 m 0.4 m

0.3 m 8 kN 14 kN

BX 0.8 m A C E G H

BY FY
300 mm


Fx : Bx = 0 B F
D

Fy : By + Fy 8 14 = 0 (kN) 400 mm 400 mm 400 mm 400 mm

+ MB : (0.4)(8) + 0.8Fy 1.2(14) = 0 
Fx : BC = 0
Solving, we get Bx = 0, By = 5.00 kN Fy = 17.00 kN.
The forces we are seeking are involved at joints B, C, D, and E. The 
Fy : AC + CE = 0
method of joints allows us to solve for two unknowns at a joint. We
need a joint with only two unknowns. Joints A and H qualify. Joint
A is nearest to the members we want to know about, so let us choose Solving, we get BC = 0,
it. Assume tension in all members. CE = 10.67 kN (T )
Joint A:
Joint B:
y
y
8 kN

AB BC
AC BE
4 x

AB
5 3
BD x

sin = 0.6 cos = 0.8 = 36.87


BY

Fx = AC + AB cos = 0

Fy = 8 AB sin = 0 We know AB = 13.33 kN BC = 0 By = 5.00 kN.
We know 3 of the 5 forces at B Hence, we can solve for the other two.

Solving, we get AC = 10.67 kN (T ) Fx : BD + BE cos AB cos = 0
AB = 13.33 kN (C) 
Fy : BC + By + BE sin + AB sin = 0
Joint C: (Again, assume all forces are in tension)
Solving, we get BD = 14.67 kN (C)
y
BE = 5.00 kN (T )

AC From Joint C, we had BC = 0


CE
x BC = 0, BD = 14.67 kN (C)
Thus
BE = 5.00 kN (T )
BC

[AC = 10.67 kN (T )]
Problem 6.17 Determine the axial forces in the mem-
B E
bers in terms of the weight W .

1m
A

D W

1m

0.8 m 0.8 m 0.8 m

Solution: Denote the axial force in a member joining two points Solving these two equations in two unknowns:
I, K by IK. The angle between member DE and the positive x axis CD = DE = 1.28W (C) , BD = 0
is = tan1 0.8 = 38.66 . The angle formed by member DB
Joint B:
with the positive x axis is 90 + . The angle formed by member AB 
with the positive x-axis is . Fx = BE AB sin BD sin = 0,
Joint E:
 BE
Fy = DE cos W = 0, from which AB = sin
= 1.28W (T )

from which DE = 1.28W (C) . Fy = AB cos BC = 0,

Fy = BE DE sin = 0, from which BC = AB cos = W (C)

from which BE = 0.8W (T )


Joint D:

Fx = DE cos + BD cos CD cos = 0,
from which BD CD = DE.

Fy = BD sin + DE sin CD sin = 0,
from which BD + CD = DE.

Problem 6.18 Consider the truss in Problem 6.17.


Each member will safely support a tensile force of 6 kN
and a compressive force of 2 kN. Use this criterion to
determine the largest weight W the truss will safely sup-
port.

Solution: From the solution to Problem 6.17, the largest ten-


sile force is in member AB, AB = 1.28W (T ), from which
6
W = 1.28 = 4.69 kN is the maximum safe load for tension.
The largest compressive forces occur in members DE and CD,
2
DE = CD = 1.28W (C), from which W = 1.28
= 1.56 kN
is the largest safe load for compression.
Problem 6.21 Each member of the truss will safely
support a tensile force of 4 kN and a compressive force 1m 1m 1m
of 1 kN. Determine the largest mass m that can safely E F
be suspended.
1m

C D
m
1m

AB

Solution: The common interior angle BAC = DCE =


EF D = CDB is = tan1 (1) = 45 .
Note cos = sin = 1 . Denote the axial force in a member 1m 1m 1m
2
joining two points I, K by IK. E
Joint F: F
 DF
Fy = W = 0, 1m
2
C
from which DF = 2W (C).
D
 DF
Fx = EF = 0, 1m
m
2
from which EF = W (T ). A
B
Joint E:
 CE
Fx = + EF = 0
2 ED
EF
CE EF CD DF
from which CE = 2W (T ).
ED
DF BD
 W
CE Joint F Joint E Joint D
Fy = ED = 0,
2 CE BC BD

CD
from which ED = W (C). AB
BC B
AC
Joint D: Joint C Joint B
 DF BD
FY = ED + = 0,
2 2 Joint B:
 BD
from which BD = 2 2W (C). Fx = AB + = 0,
2
 DF BD
FX = CD = 0, from which AB = 2W (C)
2 2
This completes the determination of the axial forces in
all nine members. The
from which CD = W (T ) maximum tensile force occurs inmember AC, AC = 2 2W (T ), from which
4
the safe load is W = = 2 = 1.414 kN. The maximum compression
Joint C: 2 2
 occurs in member BD, BD = 2 2W (C), from which the maximum safe
AC CE load is W = 1
= 0.3536 kN. The largest mass m that can be safely
Fx = + + CD = 0, 2 2
2 2
353.6
supported is m = 9.81
= 36.0 kg
from which AC = 2 2W (T )

 AC CE
Fy = + BC = 0,
2 2
from which BC = W (C)
Problem 6.26 The Howe truss helps support a roof.
800 lb
Model the supports at A and G as roller supports. De-
termine the axial forces in members AB, BC, and CD. 600 lb 600 lb
D
400 lb 400 lb
C E
8 ft
B F

A G
H I J K L

4 ft 4 ft 4 ft 4 ft 4 ft 4 ft

Solution: The strategy is to proceed from end A, choosing joints


with only one unknown axial force in the x- and/or y-direction, if
possible, and if not, establish simultaneous conditions in the unknowns. 800 lb
The interior angles HIB and HJC differ. The pitch angle is
600 lb D
  600 lb
8
Pitch = tan1 = 33.7 . 400 lb
12 C E 400 lb
8 ft
The length of the vertical members: F
B
 
8
BH = 4 = 2.6667 ft, A G
12
H I J K L
from which the angle
 
2.6667
HIB = tan1 = 33.7 . 4 ft 4 ft 4 ft 4 ft 4 ft 4 ft
4
8
CI = 8 = 5.3333 ft,
12 800 lb
600 lb 600 lb
from which the angle 400 lb 400 lb
 
5.333
IJC = tan1 = 53.1 .
4 A G
The moment about G:
4 ft 4 ft 4 ft 4 ft 4 ft 4 ft
MG = (4 + 20)(400) + (8 + 16)(600) + (12)(800) 24A = 0,
AB BH 400 lb BC
from which A = 33600
24
= 1400 lb. Check: The total load is 2800 lb. Pitch Pitch Pitch
From left-right symmetry each support A, G supports half the total 1400 lb AH AH HI AB
load. check. BH BI
The method of joints: Denote the axial force in a member joining two Joint A Joint H Joint B
points I, K by IK. 600 lb
BI CI CD
Joint A: Pitch
 Pitch IJC
Fy = AB sin P + 1400 = 0, HI IJ BC CI CJ
1400
Joint I Joint C
from which AB = sin
= 2523.9 lb (C)
p

 Joint H:
Fx = AB cos Pitch + AH = 0, 
Fy = BH = 0, or, BH = 0.
from which AH = (2523.9)(0.8321) = 2100 lb (T ) 
Fx = AH + HI = 0,

from which HI = 2100 lb (T )


Joint B:

Fx = AB cos Pitch + BC cos Pitch
+BI cos Pitch = 0,
from which BC + BI = AB
6.26 Contd.

Fy = 400 AB sin Pitch + BC sin Pitch
BI sin Pitch = 0,

400
from which BC BI = AB + sin Pitch
.

Solve the two simultaneous equations in unknowns BC, BI:

400
BI = = 360.56 lb (C),
2 sin Pitch

and BC = AB BI = 2163.3 lb (C)

Joint I:


Fx = BI cos Pitch HI + IJ = 0,

from which IJ = 1800 lb (T )


Fy = +BI sin Pitch + CI = 0,

from which CI = 200 lb (T )

Joint C:


Fx = BC cos Pitch + CD cos Pitch + CJ cos IJC = 0,

from which CD(0.8321) + CJ(0.6) = 1800


Fy = 600 CI BC sin Pitch + CD sin Pitch
CJ sin IJC = 0,

from which CD(0.5547) CJ(0.8) = 400

Solve the two simultaneous equations to obtain CJ =


666.67 lb (C),

and CD = 1682.57 lb (C)


Problem 6.30 The truss supports a 100-kN load at J.
The horizontal members are each 1 m in length. A B C D
(a) Use the method of joints to determine the axial force
in member DG. 1m
(b) Use the method of sections to determine the axial
J
force in member DG. E F G H

100 kN

Solution: (a) Start with Joint J


A B C D
DJ

1m
45

J J
E F G H
HJ
1m 1m 1m 1m
100 kN
100 kN

Fx : CD DG cos 45 + DJ cos 45 = 0

 Fy : DG sin 45 DJ sin 45 DH = 0

Fx : HJ DJ cos 45 = 0
 Solving, CD = 200 kN
Fy : DJ sin 45 100 = 0
DG = 141.4 kN (C)
Solving DJ = 141.4 kN (T ) (b) Method of Sections
HJ = 100 kN (C) CD
D
Joint H:
45
1m
DH DG

J
HJ GH H

GH H 1m
100 kN

Fx : HJ GH = 0

Fy : DH = 0

DH = 0, Fx : CD DG cos 45 GH = 0

GH = 100 kN (C) Fy : DG sin 45 100 = 0

Joint D MD : (1)GH (1)(100) = 0
CD D
Solving, GH = 100 kN (C)
x
45 CD = 200 kN (T )
45
DG = 141.4 kN (C)
DJ
DH
DG
Problem 6.36 For the Howe and Pratt trusses in Prob-
lem 6.35, determine the axial force in member HI.

Solution: Howe Section: From the Howe truss section, we see


that if we sum moments about C, we get one equation in one un-
C TCD D y
known, i.e.,
 TCI
MC = 2LEY 2LF LTHIHowe = 0,
I E
or THIHowe = 3.5F (tension). H THI x
EY
Pratt Section: From the Pratt truss section we see that summing mo- 2F
ments about D is advantageous. Hence,
 HOWE
MD = LEY LTHIPratt = 0
or THIPratt = 2.75F (tension). TCD
D y

THD I E
H THI x
EY
2F

PRATT

Problem 6.37 The Pratt bridge truss supports ve


forces F = 340 kN. The dimension L = 8 m. Use L L L L L L
the method of sections to determine the axial force in
B C D E G
member JK.
L
A H
I J K L M

F F F F F

Solution: First determine the external support forces.


L L L L L L
B C D E G
AX
L L L L L L
L
AY F F F F F HY A H
I J K L M
F = 340 kN, L = 8 M
F F F F F

Fx : Ax = 0
 = 45
Fy : Ay 5F + Hy = 0
L = 8M

+ MA : 6LHy LF 2LF 3LF 4LF 5LF = 0 F = 340 kN
Ay = 850 kN
Solving: Ax = 0,

Ay = 850 kN Fx : CD + JK + CK cos = 0
Hy = 850 kN 
Fy : Ay 2F CK sin = 0
Note the symmetry: 
Method of sections to nd axial force in member JK. + MC : L(JK) + L(F ) 2L(Ay ) = 0

B C CD
D Solving, JK = 1360 kN (T )
Also, CK = 240.4 kN (T )
CK
A I J CD = 1530 kN (C)
L L K

AY
F JK
F
Problem 6.41 The mass m = 120 kg. Use the method
of sections to determine the axial forces in members BD, 1m 1m 1m
CD, and CE. E F

1m

C D
m
1m

AB

Solution: First, nd the support reactions using the rst free body
diagram. Then use the section shown in the second free body diagram
to determine the forces in the three members.
1m 1m 1m
Support Reactions: Equilibrium equations are
 E F
Fx = AX = 0,
1m

Fy = AY + BY mg = 0, C 120 g
D
and summing moments around A,
 1m
MA = 3mg + (1)BY = 0.
AX
Thus, AX = 0, AY = 2.35kN, and BY = 3.53 kN B
Section: From the second free body diagram, the equilibrium equations
for the section are AY BY

Fx = AX + TCD + TCE cos(45 ) + TBD cos(45 ) = 0,

Fy = AY + BY + TCE sin(45 ) + TBD sin(45 ) = 0,
and, summing moments around C, 1m 1m 1m
E
 F
MC = (1)TBD cos(45 ) (1)AY + (1)AX = 0.
TCE
1m
C
D
Solving, we get TBD = 3.30 kN, 1m
TCD
TBD
TCD = 1.18 kN, B
AX
TCE = 1.66 kN. BY
AY

Problem 6.42 For the truss in Problem 6.41, use the


method of sections to determine the axial forces in mem-
bers AC, BC, and BD.

Solution: Use the support reactions found in Problem 6.41. The


free body diagram for the section necessary to nd the three unknowns 1m 1m 1m
is shown at right. The equations of equilibrium are E F

Fx = AX + TAC cos(45 ) + TBD cos(45 ) = 0, 1m
 C
Fy = AY + BY + TBC + TAC sin(45 ) + TBD sin(45 ) = 0, TAC TBC D
1m TBD
and, summing moments around B,
 AX
MB = (1)AY (1)TAC sin(45 ) = 0. BY
AY
The results are
TAC = 3.30 kN,
TBC = 1.18 kN,
and TBD = 3.30 kN.
Problem 6.45 Use the method of sections to determine
the axial force in member EF . 10 kip A

4 ft
10 kip C
B
4 ft

D E

4 ft
G
F

4 ft
I
H

12 ft

Solution: The included angle at the apex BAC is


 
12 10 kip A
= tan1 = 36.87 .
16
4 ft
The interior angles BCA, DEC, F GE, HIG are = 90 =
10 kip
53.13 . The length of the member ED is LED = 8 tan = 6 ft. C
The interior angle DEF is
B
  4 ft
4
= tan1 = 33.69 . E
LED D
The complete structure as a free body: The moment about H is MH = 4 ft
10(12) 10(16) + I(12) = 0, from which I = 280 12
= 23.33 kip.
The sum of forces: G
F

Fy = Hy + I = 0, 4 ft
from which Hy = I = 23.33 kip. I
 H
Fx = Hx + 20 = 0,
from which Hx = 20 kip. Make the cut through EG, EF , and
DE. Consider the upper section only. Denote the axial force in a 12 ft
member joining I, K by IK. The section as a free body: The sum of
the moments about E is ME = 10(4) 10(8) + DF (LED ) = 0,
from which DF = 1206
= 20 kip.
The sum of forces: F = 10 kip

Fy = EF sin EG sin DF = 0,
F = 10 kip

Fx = EF cos + EG cos + 20 = 0,
from which the two simultaneous equations: 0.5547EF + 0.8EG = E

20, and 0.8320EF 0.6EG = 20. Solve: EF = 4.0 kip (T ) DF EF EG
Problem 6.56 Consider the frame in Problem 6.55.
The cable CE will safely support a tension of 10 kN.
Based on this criterion, what is the largest downward
force F that can be applied to the frame?
F 4G
y
Solution: From the solution to Problem 6.55: E = 3
,
3 F
Gy = F A, and A = 5 F . Back substituting, E = 5 or
F = 5E, from which, for E = 10 kN, F = 50 kN

Problem 6.57 The hydraulic actuator BD exerts a 6-


kN force on member ABC. The force is parallel to BD,
and the actuator is in compression. Determine the forces A B C
on member ABC, presenting your answers as shown in
Fig. 6.35.
0.5 m
0.5 m

0.5 m

Solution: The surface at C is smooth.


Element ABC: The sum of the moments about A is
 A B C
M = (0.5)B sin 45 + (1)C = 0,
0.5 m 0.5 m
from which C = 3(0.707) = 2.121 kN . D
The sum of the forces: 0.5 m

Fy = Ay + B sin 45 + C = 0,
C
from which Ay = B (0.707) C = 2.121 kN . AY
 AX
Fx = Ax B cos 45 = 0, B 45
0.5 0.5
from which Ax = 4.24 kN m m

2.12 kN
A B C
4.24
kN 45
2.12 kN 6 kN
Problem 6.63 The tension in cable BD is 500 lb.
G E
Determine the reactions at A for cases (1) and (2).
6 in
D

6 in

A B C
300 lb
8 in 8 in
(1)

Solution: Case (a) The complete structure as a free body: The


sum of the moments about G: G E

MG = 16(300) + 12Ax = 0,
6 in
D
from which Ax = 400 lb . The sum of the forces:
 6 in
Fx = Ax + Gx = 0,
from which Gx = 400 lb. A B C

Fy = Ay 300 + Gy = 0, 300 lb
8 in 8 in
from which Ay = 300 Gy . Element GE: The sum of the mo-
(2)
ments about E:

ME = 16Gy = 0, G E G E
from which Gy = 0, and from above Ay = 300 lb. 6 in 6 in
Case (b) The complete structure as a free body: The free body diagram, D D
except for the position of the internal pin, is the same as for case (a).
The sum of the moments about G is 6 in 6 in
A

MC = 16(300) + 12Ax = 0, C A B C
300 lb 300 lb
from which Ax = 400 lb . 8 in 8 in
8 in 8 in
Element ABC: The tension at the lower end of the cable is up and to
the right, so that the moment exerted by the cable tension about point
C is negative. The sum of the moments about C: (a) (b)

MC = 8B sin 16Ay = 0, Gy
  Gx Gy Ey
noting that B = 500 lb and = tan1 68 = 36.87 , (a) 12 in
Ay Gx Ex
then Ay = 150 lb. Ax 300 lb
16 in

Ay B Cy
(b)
Cx
Ax
300 lb
8 in 8 in
Problem 6.80 The weight W = 60 kip. What is the
magnitude of the force the members exert on each other
at D?

3 ft
B C
2 ft
D

3 ft

3 ft 3 ft

Solution: Assume that a tong half will carry half the weight, and
denote the vertical reaction to the weight at A by R. The complete
structure as a free body: The sum of the forces:

Fy = R W = 0, A
from which R = W 3 ft
B C
Tong-Half ACD:
2 ft
Element AC: The sum of the moments about A:

(1) MA = 3Cy + 3Cx = 0. D 3 ft

The sum of the forces:


 R
(2) Fy = + Cy + Ay = 0, and
2

(3) Fx = Cx + Ax = 0. W
Element CD: The sum of the forces:

(4) Fx = Dx P Cx = 0, and
3 ft 3 ft
 W
(5) Fy = Dy Cy = 0.
2
The sum of the moments:
 3 Ay R R Ay
(6) MD = 2Cx 3Cy 3P + W = 0
2 2 2
Ax Ax
Element AB: The sum of the forces: By
 R Cy
(7) Fy = Ay + By = 0, and
2
 Bx Dy Cx
(8) Fx = Ax Bx = 0. Dy
Element BD: The sum of the forces: Bx Cx
Dx
 W
(9) Fy = By Dy = 0, and
2 Dx Cy
 By
(10) Fx = Bx Dx + P = 0.
P P
These are ten equations in ten unknowns. These have the solution
R = 60 kip. Check, Ax = 30 kip, Ay = 0, Bx = 30 kip, W W
By = 30 kip, Cx = 30 kip, Cy = 30 kip, Dx = 110 kip, 2 2
Dy = 0, and P = 80 kip. The magnitude of the force the
members exert on each other at D is D = 110 kip.
Problem 6.85 The mechanism is used to weigh mail.
A package placed at A causes the weighted point to rotate
through an angle . Neglect the weights of the members A
except for the counterweight at B, which has a mass of
4 kg. If = 20 , what is the mass of the package at A?

100 100
mm mm

30

Solution: Consider the moment about the bearing connecting the


motion of the counter weight to the motion of the weighing plat-
form. The moment arm of the weighing platform about this bearing
is 100 cos(30 ). The restoring moment of the counter weight is
100 mg sin . Thus the sum of the moments is A

M = 100 mB g sin 100 mA g cos(30 ) = 0.
Dene the ratio of the masses of the counter weight to the mass of the
package to be RM = m B
mA
. The sum of moments equation reduces to
 100
M = RM sin cos(30 ) = 0,
mm 100
cos(30)
from which RM = sin
= 2.8794, and the mass of the pack- mm
age is mA = R4 = 1.3892 = 1.39 kg
M B

30


100 N
Problem 6.89 Determine the force exerted on the bolt
by the bolt cutters.
A
75 40 mm
mm C 55 mm
B
D

90 mm 60 mm 65 mm 300 mm
100 N

Solution: The equations of equilibrium for each of the members


will be developed. 100 N
Member AB: The equations of equilibrium are:
 A
FX = AX + BX = 0, 75 40 mm
 mm
B C 55 mm
FY = AY + BY = 0,
D

and MB = 90F 75AX 425(100) = 0 F
Member BD: The equations are

FX = BX + DX = 0,
90 mm 60 mm 65 mm 300 mm
 100 N
FY = BY + DY + 100 = 0,

and MB = 15DX + 60DY + 425(100) = 0. 100 N
AX AY
Member AC: The equations are F

FX = AX + CX = 0, 75 mm
40 mm

 A B 55 mm
FY = AY + CY + F = 0,
 BX BY
and MA = 90F + 125CY + 40CX = 0.
Member CD: The equations are:

FX = CX DX = 0, 90 mm 60 mm 65 mm 300 mm

FY = CY DY = 0. AY
AX CY
Solving the equations simultaneously (we have extra (but compatible)
40 mm
equations, we get F = 1051 N, AX = 695 N, AY = 1586 N, 75 mm C
55 mm X
BX = 695 N, BY = 435 N, CX = 695 N, CY = 535 N, C
D
DX = 695 N, and Dy = 535 N F

90 mm 60 mm 65 mm 300 mm

BY

B DX
BX D
DY

60 mm 65 mm 300 mm
100 N

CY

C CX

DX
D
DY

Das könnte Ihnen auch gefallen